matheraum.de
Raum für Mathematik
Offene Informations- und Nachhilfegemeinschaft

Für Schüler, Studenten, Lehrer, Mathematik-Interessierte.
Hallo Gast!einloggen | registrieren ]
Startseite · Forum · Wissen · Kurse · Mitglieder · Team · Impressum
Forenbaum
^ Forenbaum
Status Mathe
  Status Schulmathe
    Status Primarstufe
    Status Mathe Klassen 5-7
    Status Mathe Klassen 8-10
    Status Oberstufenmathe
    Status Mathe-Wettbewerbe
    Status Sonstiges
  Status Hochschulmathe
    Status Uni-Analysis
    Status Uni-Lin. Algebra
    Status Algebra+Zahlentheo.
    Status Diskrete Mathematik
    Status Fachdidaktik
    Status Finanz+Versicherung
    Status Logik+Mengenlehre
    Status Numerik
    Status Uni-Stochastik
    Status Topologie+Geometrie
    Status Uni-Sonstiges
  Status Mathe-Vorkurse
    Status Organisatorisches
    Status Schule
    Status Universität
  Status Mathe-Software
    Status Derive
    Status DynaGeo
    Status FunkyPlot
    Status GeoGebra
    Status LaTeX
    Status Maple
    Status MathCad
    Status Mathematica
    Status Matlab
    Status Maxima
    Status MuPad
    Status Taschenrechner

Gezeigt werden alle Foren bis zur Tiefe 2

Navigation
 Startseite...
 Neuerdings beta neu
 Forum...
 vorwissen...
 vorkurse...
 Werkzeuge...
 Nachhilfevermittlung beta...
 Online-Spiele beta
 Suchen
 Verein...
 Impressum
Das Projekt
Server und Internetanbindung werden durch Spenden finanziert.
Organisiert wird das Projekt von unserem Koordinatorenteam.
Hunderte Mitglieder helfen ehrenamtlich in unseren moderierten Foren.
Anbieter der Seite ist der gemeinnützige Verein "Vorhilfe.de e.V.".
Partnerseiten
Dt. Schulen im Ausland: Mathe-Seiten:Weitere Fächer:

Open Source FunktionenplotterFunkyPlot: Kostenloser und quelloffener Funktionenplotter für Linux und andere Betriebssysteme
StartseiteMatheForenTrigonometrische FunktionenKanonenbeschuss
Foren für weitere Schulfächer findest Du auf www.vorhilfe.de z.B. Philosophie • Religion • Kunst • Musik • Sport • Pädagogik
Forum "Trigonometrische Funktionen" - Kanonenbeschuss
Kanonenbeschuss < Trigonometr. Fktn < Analysis < Oberstufe < Schule < Mathe < Vorhilfe
Ansicht: [ geschachtelt ] | ^ Forum "Trigonometrische Funktionen"  | ^^ Alle Foren  | ^ Forenbaum  | Materialien

Kanonenbeschuss: Frage (beantwortet)
Status: (Frage) beantwortet Status 
Datum: 10:52 Di 09.11.2010
Autor: newflemmli

Aufgabe
Ein Schiff, das als Schiffszeichen zwei goldene Dolche trägt, ankert zwecks Alkoholbeschaffung vor einer Insel.

Heimtückisch hat sich bei Nacht und Nebel auch ein feindliches Schiff der Insel genähert; der lauert nun versteckt hinter einem Hügel auf der Gegenseite der Insel in 1732 Fuss Entfernung vom ursprünglichen Schiff. Der grausame Pirat Roberts plant, mit der grossen Zwölfpfünder (Maximale Schussweite m = 2000 Fuss) dem Rumpf des zur Beladung haltenden Schiffes über den Hügel hinweg einen Volltreffer zu versetzen, um sie danach in aller Ruhe entern zu können.
a) Das Azimut und die Entfernung sind bekannt - doch in welchem Winkel muss die Kanone abgefeuert werden, um das verlockende Ziel über den 563 Fuss hohen Inselberg hinweg zu treffen?

also für y (der Wurfweite) gilt (glaub ich zumindest) doch:

[mm] y=t*v0*cos(\alpha) [/mm]

m.... maximale "Wurfweite" ist mir bekannt
Wurfweite w...... 1732 Fuss

und für die Wurfhöhe:
[mm] z=t*v0*sin(\alpha) [/mm]
________________________________________________________

(1) vernachlässigen wir h, sprich sei die Höhe des "Piratenschiffes" h=0.

Wir haben eine Auswahl von Formel bekommen und sollen zeigen welche gilt für

(a) die maximale Wurfhöhe δ
a) δ(α) = cos(mα), b) δ(α) = m sin(2α), d) δ(α) = 2m sin(α), e) δ(α) = mβ(α) cos(2α).

Wie kann ich das herleiten wenn doch in der Formel oben ein v ist? oder muss ich mit einer anderen Formel arbeiten, wenn ja, dann bitte welche

Danke im Voraus
newflemmli

        
Bezug
Kanonenbeschuss: Antwort
Status: (Antwort) fertig Status 
Datum: 11:21 Di 09.11.2010
Autor: leduart

Hallo
die maximale Wurfweite (ohne Reibung) erreicht man wenn der Abschusswinkel 45° ist. Das habt ihr sicher schon gemacht.
deshalb kannst  du aus m die Anfangsgeschw. [mm] v_0 [/mm] ausrechnen.
erst wenn du die hast, kannst du weiter rechnen.
d,h, du musst den Winkel ausrechnen unter dem man mit dem [mm] v_0 [/mm] die 1732 Fuss
erreicht, und dazu die maximale Höhe.
dein y ist richtig, dein z nicht, in y Richtung ist die Geschw konstant, in z-Richtung aber nicht! ausserdem steht da ja noch ne unbekannte Zeit.
Am besten du schreibst erstmal [mm] v_y(t), [/mm] y(t) und [mm] v_z(t), [/mm] z(t) auf.
dann kann man losrechnen.
direkt sehen solltest du dass cos(mα) sicher falsch ist. denn cos(Länge) gibts nicht. und [mm] m*\alpha [/mm] wär ne Länge.
Gruss leduart


Bezug
                
Bezug
Kanonenbeschuss: Frage (beantwortet)
Status: (Frage) beantwortet Status 
Datum: 11:39 Di 09.11.2010
Autor: newflemmli

Hmm aber wie komme ich auf das vo

1732 = t * vo * cos(45)

ich kenne dich t nicht?
hilft mir v=s/t irgendwie weiter?


und für z fehlt mir die "Fallbewegung" oda? also einfach - g/2 * [mm] t^2? [/mm]

Bezug
                        
Bezug
Kanonenbeschuss: Antwort
Status: (Antwort) fertig Status 
Datum: 11:49 Di 09.11.2010
Autor: leduart

Hallo
1. hast du [mm] v_0 [/mm] bestimmt?
2. warum nicht die vier Gesetze ,die ich gesagt habe aufschreiben?
3. du kannst die Zeit ausrechnen wenn du z(t) kennst. was ist das bei 1732?
Gruss leduart


Bezug
                                
Bezug
Kanonenbeschuss: Frage (beantwortet)
Status: (Frage) beantwortet Status 
Datum: 12:16 Di 09.11.2010
Autor: newflemmli

also ich schreib es jetzt so:

Die 4 Gesetze:
Wurfhöhe h = v0 * sin (a) - 0,5g * [mm] t^2 [/mm]
         v0,h = vo * sin (a)

Wurfweite w = v0 * sin (a) * t
         v0,w = v0 * cos(a)


mit t=w/(v*cos (a)), in h eingesetzt ergibt sich:

h= v0 * sin(a) * [mm] \bruch{w}{vo*cos(a)} [/mm] - [mm] \bruch{w^2}{vo^2*cos(a)^2} [/mm] * 0,5g

das bekomme ich dann raus.

Frage: nur wie hilft mir das t0 zu bestimmen, ich versehe es einfach nicht



Bezug
                                        
Bezug
Kanonenbeschuss: Antwort
Status: (Antwort) fertig Status 
Datum: 13:11 Di 09.11.2010
Autor: MathePower

Hallo newflemmli,

> also ich schreib es jetzt so:
>  
> Die 4 Gesetze:
>  Wurfhöhe h = v0 * sin (a) - 0,5g * [mm]t^2[/mm]
>           v0,h = vo * sin (a)
>  
> Wurfweite w = v0 * sin (a) * t
>           v0,w = v0 * cos(a)
>  
>
> mit t=w/(v*cos (a)), in h eingesetzt ergibt sich:
>  
> h= v0 * sin(a) * [mm]\bruch{w}{vo*cos(a)}[/mm] -
> [mm]\bruch{w^2}{vo^2*cos(a)^2}[/mm] * 0,5g
>  
> das bekomme ich dann raus.
>  
> Frage: nur wie hilft mir das t0 zu bestimmen, ich versehe
> es einfach nicht
>  

[mm]t_{0}[/mm] kannst Du aus dieser Gleichung nicht bestimmen,
wohl aber [mm]v_{0}[/mm]

Dazu mußt Du wissen, daß für die maximale Wurfweite
die Wurfhöhe h gleich 0 ist.

Löse also

[mm]0= v0 * sin(a) * \bruch{w}{vo*cos(a)} - \bruch{w^2}{vo^2*cos(a)^2} * 0,5g[/mm]

nach [mm]v_{0}[/mm] auf.


Gruss
MathePower

Bezug
                                                
Bezug
Kanonenbeschuss: Frage (beantwortet)
Status: (Frage) beantwortet Status 
Datum: 13:37 Di 09.11.2010
Autor: newflemmli

also ich rechne in SI Einheiten

Maximale Wurfweite = 2000 Fuss [mm] \gdw [/mm] 610 Meter
und Alpha = 45° wenn max. Wurfweite

mit h=0 beim "Endpunkt" nach 610m
w=610m
g=10

610 tan(a) - 0,5 * 10 * [mm] \bruch{610^2}{(cos(a))^2*v^2} [/mm] = 0
mit a = 45

ergibt sich für mich ein v [mm] \approx [/mm] 82,603 kann das sein?



Bezug
                                                        
Bezug
Kanonenbeschuss: Antwort
Status: (Antwort) fertig Status 
Datum: 14:05 Di 09.11.2010
Autor: Steffi21

Hallo, du hast aber die Aufgabe inhaltlich noch nicht verstanden, du hast die Anfangsgeschwindigkeit berechnet für die Annahme, deine Kanone schießt 2000 ft (609,6m) unter einem Abwurfwinkel von [mm] 45^{0}, [/mm] damit triffst du aber das andere Schiff unter Garantie nicht, es ist nur 1732ft (527,9m) entfernt, verstanden, unabhängig davon, ist dein Ergebnis falsch, die Anfangsgeschwindigkeit wäre unter deiner Annahem [mm] v_0=78,1\bruch{m}{s}, [/mm] du sollst also nur 527,9m weit schießen, jetzt kommt aber noch ein Problem, du mußt über den Berg schießen, dahinter steckt die Wurfhöhe, Steffi

Bezug
                                                                
Bezug
Kanonenbeschuss: wer hat sich nun geirrt ...
Status: (Mitteilung) Reaktion unnötig Status 
Datum: 14:46 Di 09.11.2010
Autor: Al-Chwarizmi


> Hallo, du hast aber die Aufgabe inhaltlich noch nicht
> verstanden, du hast die Anfangsgeschwindigkeit berechnet
> für die Annahme, deine Kanone schießt 2000 ft (609,6m)
> unter einem Abwurfwinkel von [mm]45^{0},[/mm] damit triffst du aber
> das andere Schiff unter Garantie nicht, es ist nur 1732ft
> (527,9m) entfernt, verstanden, unabhängig davon, ist dein
> Ergebnis falsch, die Anfangsgeschwindigkeit wäre unter
> deiner Annahem [mm]v_0=78,1\bruch{m}{s},[/mm] du sollst also nur
> 527,9m weit schießen, jetzt kommt aber noch ein Problem,
> du mußt über den Berg schießen, dahinter steckt die
> Wurfhöhe, Steffi


Hallo Steffi,

ich befürchte, dass hier du dich etwas geirrt hast. Die
Angabe mit der maximal überhaupt möglichen Schussweite
von 2000 ft braucht man für die vorbereitende Aufgabe,
in welcher erst einmal die Geschwindigkeit [mm] v_0 [/mm] berechnet
werden muss, mit welcher die Kanonenkugel die Mündung
des Kanonenrohrs verlässt.
Unterschiedliche Schussweiten kann man ja dann nur
dadurch erzielen, indem man den Neigungswinkel [mm] \alpha [/mm]
des Kanonenrohrs entsprechend einstellt - und nicht
dadurch, dass man die Kugel mit unterschiedlichen Start-
geschwindigkeiten abschießt, was natürlich durch unter-
schiedlich große Sprengstoffladungen auch realisiert
werden könnte.
Die zweite Aufgabe (nach der Berechnung von [mm] v_0) [/mm] ist
dann die Suche nach jenem (oder jenen) [mm] \alpha, [/mm] für welche(s)
die Wurfweite gleich 1732 ft wird.
Eine dritte Überlegung schließt sich dann noch an.


LG     Al-Chw.


Bezug
                                                                        
Bezug
Kanonenbeschuss: Frage (beantwortet)
Status: (Frage) beantwortet Status 
Datum: 15:17 Di 09.11.2010
Autor: Steffi21

Hallo Al-Chwarizmi, ich wollte in meiner Überlegung [mm] v_0 [/mm] und [mm] \alpha [/mm] ändern, so habe ich die Aufgabe interpretiert, offenbar ist die Aufgabe so zu verstehen, [mm] v_0 [/mm] ist eine feste Größe, ich kann die Wurfweite nur über [mm] \alpha [/mm] ändern, was meint ihr zu meiner Interpretation? Wenn ich falsch liege, sorry an den Aufgabensteller, Steffi

ich habe mal mit FunkyPlot "gespielt", habe [mm] v_0 [/mm] und [mm] \alpha [/mm] variert, in der Annahme mir steht [mm] v_0=77,3\bruch{m}{s} [/mm] maximal zur Verfügung, die ich aber nicht ausnutzen muß, die rote wurfparabel ist natürlich nicht möglich, die Höhe vom Berg wird nicht erreicht

[Dateianhang nicht öffentlich]




Dateianhänge:
Anhang Nr. 1 (Typ: png) [nicht öffentlich]
Bezug
                                                                                
Bezug
Kanonenbeschuss: Mitteilung
Status: (Mitteilung) Reaktion unnötig Status 
Datum: 15:36 Di 09.11.2010
Autor: newflemmli

Es ist tatsächlich so gemeint das vo eine konstante Größe ist ^^. wir dürfen das sogar annehmen das v0=const., sowie h=0 im Anfangspunkt

Bezug
                                                                                
Bezug
Kanonenbeschuss: Mitteilung
Status: (Mitteilung) Reaktion unnötig Status 
Datum: 17:25 Di 09.11.2010
Autor: newflemmli

und mit welcher formel spielst du herum? was ist denn nun genau meine Formel für meine Wurfparabel?

Bezug
                                                                                
Bezug
Kanonenbeschuss: Antwort
Status: (Antwort) fertig Status 
Datum: 18:42 Di 09.11.2010
Autor: leduart

Hallo Steffi
Piraten haben diese altmodischen Kanonen, aus denen die Kugeln immer gleich schnell rausfliegen, auf jeden Fall können sie [mm] v_0 [/mm] nicht regeln. wenn sie also kürzer s hiessen wollen müssen sie steiler (oder flacher)  als 45° schiessen. gesucht ist der größere Winkel, mit dem sie 528m erreichen. die maximale Höhe ergibt sich dan u nd reicht, oder eben nicht.
Gruss leduart


Bezug
                                                                                        
Bezug
Kanonenbeschuss: Mitteilung
Status: (Mitteilung) Reaktion unnötig Status 
Datum: 18:55 Di 09.11.2010
Autor: Steffi21

Hallo  leduart, na gut

>  Piraten haben diese altmodischen Kanonen, aus denen die
> Kugeln immer gleich schnell rausfliegen, auf jeden Fall
> können sie [mm]v_0[/mm] nicht regeln.
>  

wollte die Aufgabe eben etwas moderner betrachten, [mm] v_0 [/mm] ist regelbar, dass bei [mm] v_0=const. [/mm] es nur über den winkel geht, ist mir schon klar,

Steffi

Bezug
                                                        
Bezug
Kanonenbeschuss: Antwort
Status: (Antwort) fertig Status 
Datum: 14:34 Di 09.11.2010
Autor: Al-Chwarizmi


> also ich rechne in SI Einheiten
>  
> Maximale Wurfweite = 2000 Fuss [mm]\gdw[/mm] 610 Meter     [ok]
>  und Alpha = 45° wenn max. Wurfweite   [ok]
>  
> mit h=0 beim "Endpunkt" nach 610m
>  w=610m
>  g=10
>  
> 610 tan(a) - 0,5 * 10 * [mm]\bruch{610^2}{(cos(a))^2*v^2}[/mm] = 0
>  mit a = 45
>  
> ergibt sich für mich ein v [mm]\approx[/mm] 82,603 kann das sein?


Hallo nf,

Ich habe für die Wurfweite  W die Formel   [mm] W=\frac{{v_0}^2}{g}*sin(2\,\alpha) [/mm]  
hergeleitet und für g den Wert 9.81 [mm] m/s^2 [/mm] verwendet und bin
damit auf die Kanonenschussgeschwindigkeit  [mm] v_0\approx [/mm] 77.3  m/s  
gekommen.


LG   Al-Chw.

Bezug
                                                                
Bezug
Kanonenbeschuss: Frage (beantwortet)
Status: (Frage) beantwortet Status 
Datum: 15:34 Di 09.11.2010
Autor: newflemmli

habs mit den genauen Werten nachgerechnet und komme auf das richtige v0 :D,

Jetzt muss ich mir nur noch überlegen wie ich den winkel einstellen muss, und dafür brauch ich die h Funktion oder?

Nur wie stelle ich sicher, dass ich in der Entfernuzt aung auch tatsächlich über den Berg schieße?

Vorallem stört mich nun wieder das bösartige t, denn die Wurfhöhe hängt ja von t ab, weil die Erdanziehung die Beschleunigung verändert oder?

Aber ich hab noch immer nicht die Antwort auf die erste Frage, denn welche formel stimmt den nun?
den ich muss das in abhängigkeit von m darstellen. Intuitiv hätte ich jetzt auf m*sin(2a) getippt.

und für die maximale Schusshöhe hätte ich dann gesagt:

Wurfweite(a) / 2 = Wurfmaxhöhe(a), weil es ja eine parabel ist oder? Kann mir wer den Ansatz zeigen ? langsam versteh ich es immer besser :D

Bezug
                                                                        
Bezug
Kanonenbeschuss: Antwort
Status: (Antwort) fertig Status 
Datum: 16:36 Di 09.11.2010
Autor: Al-Chwarizmi


> Jetzt muss ich mir nur noch überlegen wie ich den winkel
> einstellen muss, und dafür brauch ich die h Funktion
> oder?
>  
> Nur wie stelle ich sicher, dass ich in der Entfernuzt aung
> auch tatsächlich über den Berg schieße?

Am besten denkt man sich die Sache zuerst ganz ohne
den Berg, der der Kugel allenfalls im Weg stehen könnte.
  

> Vorallem stört mich nun wieder das bösartige t, denn die
> Wurfhöhe hängt ja von t ab, weil die Erdanziehung die
> Beschleunigung verändert oder?

was ist denn da bösartig ?

Aufgelöst in die horizontale und die vertikale Komponente:

    $\ x(t)\ =\ [mm] v_x*t$ [/mm]

    $\ y(t)\ =\ [mm] v_y*t-\frac{g}{2}*t^2$ [/mm]

Damit kann man alles Weitere, was noch interessiert, in
der Art einer gewöhnlichen Kurvendiskussion untersuchen.

> Aber ich hab noch immer nicht die Antwort auf die erste
> Frage, denn welche formel stimmt den nun?
>  den ich muss das in abhängigkeit von m darstellen.
> Intuitiv hätte ich jetzt auf m*sin(2a) getippt.
>  
> und für die maximale Schusshöhe hätte ich dann gesagt:
>  
> Wurfweite(a) / 2 = Wurfmaxhöhe(a), weil es ja eine parabel
> ist oder? Kann mir wer den Ansatz zeigen ? langsam versteh
> ich es immer besser :D

Ich würde dir empfehlen, die Frage a) am Schluss zu betrachten,
wenn du das Ganze mit deinen eigenen Gedanken durchdrungen
hast ...

Mich stört an der Aufgabenstellung ein wenig das m, das zuerst
als konstant vorgegebene maximale Schussweite der Kanone
eingeführt wurde. Nun kann die Kanone ihre maximale Weite nur
beim Abschusswinkel  [mm] \alpha [/mm] = 45°  erreichen.  
Für die maximal erreichte Höhe bei einem Abschusswinkel [mm] \alpha [/mm]
habe ich erhalten:   $\ [mm] y_{max}\ [/mm] =\ [mm] \frac{{v_0}^2}{2\,g}*sin^2(\alpha)$ [/mm]

Dies kann ich nun mit keinem der angegebenen  [mm] \delta [/mm] - Werte identifizieren.
Oder war da vielleicht mit m doch plötzlich was anderes gemeint ? ...


LG     Al-Chw.

Bezug
                                                                        
Bezug
Kanonenbeschuss: Antwort
Status: (Antwort) fertig Status 
Datum: 19:04 Di 09.11.2010
Autor: leduart

Hallo
um Klarheit zu verbreiten noch mal die Formeln:
[mm] v_x(t)=v_0*cos(\alpha) [/mm]
[mm] x(t)=v_0*cos(\alpha)*t [/mm]
[mm] v_y(t)=v_0*sin(\alpha)-gt [/mm]
[mm] y=v_0*sin(\alpha)*t-g/2t^2 [/mm]
Höhe ist erreicht bei [mm] v_y(th)=0 [/mm]
also [mm] v_0*sin(\alpha)-gt_h=0 t_h=1/g*v_0*sin(\alpha) [/mm]
da man ohne Reibung arbeitethat man ne echte Parabel, die Kugel braucht solange rauf wie runter, deshalb [mm] t_w=2t_h [/mm]
und damit [mm] x(t_w)=w=v_o*cos(\alpha)*2/g*v_0*sin(\alpha) [/mm]
und mit [mm] 2sin(\alpha)*cos(\alpha)=sin(2*\alpha) [/mm]
[mm] w=*v_0^2/g*sin(2\alpha) [/mm]
mit w=m [mm] 2\alpha)=90° [/mm] hat man [mm] v_0^2=610m*9,81m/s^2 [/mm]
[mm] v_0=73,4m/s [/mm]
jetzt will man w=528m
also [mm] 528m=610m*sin(2\alpha) [/mm] daraus [mm] 2\alpha [/mm]
Vorsicht, 2 Werte, der TR liefer nur einen (ca60°) aber man muss den grösseren [mm] 180-2\alppa [/mm] nehmen.
dazu rechnet man dann mit der Formel oben [mm] t_h [/mm] aus, darau [mm] y_h=h [/mm] und das reicht oder nicht, um über den Hügel zu kommen.
Aber so wie ich oben sollte man erstmal sehr genau alle benötigten gleichungen aufschreiben und nicht mit irgendwelchen bunt rausgesuchten rumfummeln.
Gruss leduart



Bezug
        
Bezug
Kanonenbeschuss: technische Fragen ... (off t.)
Status: (Mitteilung) Reaktion unnötig Status 
Datum: 12:09 Di 09.11.2010
Autor: Al-Chwarizmi

Hallo,

mir ist nicht ganz klar, wie die Piraten es bewerkstelligen,
bei Nacht und Nebel die Position des anderen Schiffs, das
hinter dem Berg, auf der anderen Seite der Insel ankert,
auf den Fuß genau zu ermitteln. Sie müssen offenbar
auf dem eigenen Schiff GPS haben (das ist bei modernen
Piraten nichts aussergewöhnliches), sie müssen aber wohl
am anderen Schiff ebenfalls ein GPS-Gerät mit Funksender
angebracht haben, um dessen genaue Position übermittelt
zu bekommen ...

Um wirklich genau zu beurteilen, ob eine Kanonenkugel
wirklich von Schiff A über den Berg zu Schiff B gelangen
kann, müsste man noch genaueres wissen über das
vertikale Profil des Berges in der Ebene der Schussbahn.

Übrigens noch ein wichtiger Hinweis für die Rechnungen:
die physikalischen Rechnungen sollte man sinnvollerweise
in SI-Einheiten durchführen, d.h. die in Fuß angegebenen
Distanzen müssen in Meter umgerechnet werden, sonst
geht das z.B. mit dem g (Grav.konstante) schief !


(man könnte natürlich auch in einem Fuß-Sekunden-
system rechnen und dazu zuerst den Zahlenwert der
entsprechenden g-Konstante berechnen)


LG    Al-Chw.




Bezug
        
Bezug
Kanonenbeschuss: Mitteilung
Status: (Mitteilung) Reaktion unnötig Status 
Datum: 18:12 Di 09.11.2010
Autor: Sax

Hi,

kann es sein, dass in deinem Katalog möglicher Lösungen c) fehlt ?
Vielleicht c)   [mm] \delta (\alpha) [/mm] = [mm] \bruch{m}{2}*sin^2\alpha [/mm]  ?

Gruß Sax.

Bezug
                
Bezug
Kanonenbeschuss: Mitteilung
Status: (Mitteilung) Reaktion unnötig Status 
Datum: 18:25 Di 09.11.2010
Autor: newflemmli

c fehlt, ist aber :   m * [mm] cos(a^2) [/mm]

Bezug
Ansicht: [ geschachtelt ] | ^ Forum "Trigonometrische Funktionen"  | ^^ Alle Foren  | ^ Forenbaum  | Materialien


^ Seitenanfang ^
www.matheraum.de
[ Startseite | Forum | Wissen | Kurse | Mitglieder | Team | Impressum ]